Saltar ao contido principal
Microsoft
|
Math Solver
Resolver
Prácticas
Xogar
Temas
Pre-álxebra
Media
Modo
Maior factor común
Múltiplos menos comúns
Orde de operacións
Fraccións
Fraccións mixtas
Primeira factorización
Expoñentes
Radicais
Álxebra
Combinar termos como
Solución para unha variable
Factor
Expandir
Avaliar fraccións
Ecuacións lineares
Ecuacións cuadráticas
Desigualdades
Sistemas de ecuacións
Matrices
Trigonometría
Simplificación
Avaliación
Gráficos
Resolver ecuacións
Cálculo
Derivadas
Integrais
Límites
Entradas de álxebra
Entradas trigonometrías
Entradas de cálculo
Entradas de matriz
Resolver
Prácticas
Xogar
Temas
Pre-álxebra
Media
Modo
Maior factor común
Múltiplos menos comúns
Orde de operacións
Fraccións
Fraccións mixtas
Primeira factorización
Expoñentes
Radicais
Álxebra
Combinar termos como
Solución para unha variable
Factor
Expandir
Avaliar fraccións
Ecuacións lineares
Ecuacións cuadráticas
Desigualdades
Sistemas de ecuacións
Matrices
Trigonometría
Simplificación
Avaliación
Gráficos
Resolver ecuacións
Cálculo
Derivadas
Integrais
Límites
Entradas de álxebra
Entradas trigonometrías
Entradas de cálculo
Entradas de matriz
Básico
Álxebra
trigonometría
Cálculo
Estatísticas
Matrices
Personaxes
Calcular
\infty
Quiz
Limits
5 problemas similares a:
\lim_{ x \rightarrow 0 } \frac{1}{x^2}
Problemas similares da busca web
Showing that the \lim_{x\to 0}\frac{1}{x^2} does not exist
https://math.stackexchange.com/q/1579837
Suppose that the limit exists and equals c\in\mathbb{R}. Then for e.g. \epsilon>1 some \delta>0 must exist with \left|x\right|<\delta\implies\left|\frac{1}{x^{2}}-c\right|<1. However, if we ...
Applying L'Hopital's rule to \lim\limits_{x \to 0}\frac{2}{x^2}
https://math.stackexchange.com/questions/502024/applying-lhopitals-rule-to-lim-limits-x-to-0-frac2x2
In order to use the 0/0 case of L'Hospital's rule, we require that both the numerator and the denominator tend to 0 at the appropriate point. The numerator does not tend to 0.
Is this piece-wise function continuous, and why?
https://math.stackexchange.com/questions/2411697/is-this-piece-wise-function-continuous-and-why
If we look at the behaviour as x approaches zero from the right, the function looks like this: \begin{matrix}x & f(x) = \frac{1}{x^2} \\ 1 & 1 \\ 0.1 & 100 \\ 0.01 & 10000 \\ 0.001 & 1000000 \\ 0.0001 & 100000000\end{matrix} ...
Manipulating \lim\limits_{x \to 0}{\frac{\sqrt{x+\sqrt{x}}}{x^n}}
https://math.stackexchange.com/questions/2177214/manipulating-lim-limits-x-to-0-frac-sqrtx-sqrtxxn
If \lim\limits_{x \to 0}{\frac{\sqrt{x+\sqrt{x}}}{x^n}} = c for some c\neq 0, then \lim\limits_{x \to 0}{\frac{x+\sqrt{x}}{x^{2n}}} =c^2. Now, let \sqrt{x}=t. We then wish to find n such ...
Limit of \frac{f'(x)}{g'(x)} & g'(x) \neq 0 in Hypotheses of L'Hospital's rule.
https://math.stackexchange.com/q/110408
When we write things like \lim_{x\to a}h(x) = \lim_{x\to a}H(x) we usually mean "if either limit exists, then they both do and they are equal; if either limit does not exist, then neither limit ...
How do we calculate the Right and Left Hand Limit of 1/x?
https://math.stackexchange.com/questions/762599/how-do-we-calculate-the-right-and-left-hand-limit-of-1-x
\mathbf{Definition} : \boxed{ \lim_{x \to a^+ } f(x) = \infty } means that for all \alpha > 0, there exists \delta > 0 such that if 0<x -a < \delta, then f(x) > \alpha \mathbf{Example} ...
Máis Elementos
Compartir
Copia
Copiado a portapapeis
Problemas similares
\lim_{ x \rightarrow 0 } 5
\lim_{ x \rightarrow 0 } 5x
\lim_{ x \rightarrow 0 } \frac{2}{x}
\lim_{ x \rightarrow 0 } \frac{1}{x^2}
Volver arriba